You are on page 1of 3

Errata for A&J Questions Bank for MFE/3F

3rd Edition
Errata for A&J Questions Bank SOA Exam MFE/ CAS Exam 3F

1. Part 1 Q26: Naomi purchases dollar-denominated options to secure the company against
the strengthening of US dollar.

2. Part 1 Q 27: The date today is April 1, 2009.

3. Part 8 Q40: The solution is incorrect. The correct solution is shown below:

Var  ln W 

 Var ln  S 1  S  2     Var ln  S 1  S  2   


0.5 1
  4
1
 
 Var ln S  0  e   Z1  S  0  e 2   2 Z 2 
4
1 2
 Var     Z1  2  2 Z 2   Var  Z1  2Z 2 
4 4
2 2
 Var  Z1   4Var  Z 2    1  4
4  4
5 2 5
  0.15 
2

4 4
= 0.028125

4. Part 9 Q7: The solution is incorrect. There should be a positive payoff at node ud (or du):

The call option with strike price of $0.92 has a positive payoff only when the interest rate is
lower than 0.086957. If this is the case, both ud and dd nodes have positive payoff of:

1 1
 0.92  0.005926 and  0.92  0.023396
1.08 1.06

respectively. Discounting the payoff:

 1  1 
Path ud: 0.005926     0.004943
 1.11  1.08 
 1  1 
Path du: 0.005926     0.005128
 1.07  1.08 
 1  1 
Path dd: 0.023396     0.020628
 1.07  1.08 

The call option premium is therefore


 0.004943 0.6 0.4   0.005128 0.6 0.4   0.020628 0.4
2
 0.05718

5. Part 9 Q8: The answer should be Re0.2 = 0.108237e0.2 = 0.132201

Copyright © 2010 A&J, 3rd Edition |

2
Errata for A&J Questions Bank SOA Exam MFE/ CAS Exam 3F

6. Part 9 Q18: The call option premium should be multiplied by 100, since the question is
asking for the cost of 100 call options. Answer = 5.08

7. Part 9 Q19: The question should give the following volatilities:

 The volatility of a 1-year forward 1-year bond is 12%


 The volatility of a 2-year forward 1-year bond is 14%
 The volatility of a 1-year forward 2-year bond is 13%

The solution remains the same.

8. Part 4 Q26: The solutions for simultaneous equations are incorrect. They should be:

X B  2 , X C  1 and therefore X A  0.1

Hence, the hedging cost is  6  0.1  2  4    5  3.6

9. Part 4 Q28: the correct solution is:

Since rho is involved, bonds can be considered to hedge the derivatives.

Let N and M be the number of stocks and bonds needed for the hedging respectively.

Delta  N  S  M  B  0.78  N  M  0  N  0.78

B
Rho  N  S  M  B  0.2  N  0   M
r
  rT
 0.2  M e  TMe  rT   Me 0.03
r
 M  0.2061

 
As such, the hedging cost is 0.78 10   0.2061 e0.03  7.6

10. Part 3 Q35: The last sentence of the solution is:

Hence the profit made is 9.7839 – 5.6755 e0.05(0.25) = 4.0370

11. Part 9 Q40: The answer letter is wrong. It should be C instead of B.


12. Part 5 Q30: In the solution, the correlated variance should be 0.3175 instead of 0.3863.

Copyright © 2010 A&J, 3rd Edition |

You might also like